Difference between revisions of "2024 AMC 10A Problems/Problem 5"
(Created page with "A user attempted to look at 2024 AMC 10A Problem 5. What is the probability that they can see the question before the actual test date? (A) 0% (B) 0% (C) 0% (D) 0% (E) 0%") |
MRENTHUSIASM (talk | contribs) |
||
(19 intermediate revisions by 13 users not shown) | |||
Line 1: | Line 1: | ||
− | + | {{duplicate|[[2024 AMC 10A Problems/Problem 5|2024 AMC 10A #5]] and [[2024 AMC 12A Problems/Problem 4|2024 AMC 12A #4]]}} | |
+ | == Problem == | ||
+ | What is the least value of <math>n</math> such that <math>n!</math> is a multiple of <math>2024</math>? | ||
− | (A) | + | <math>\textbf{(A) } 11\qquad\textbf{(B) } 21\qquad\textbf{(C) } 22\qquad\textbf{(D) } 23\qquad\textbf{(E) } 253</math> |
+ | |||
+ | == Solution== | ||
+ | Note that <math>2024=2^3\cdot11\cdot23</math> in the prime factorization. Since <math>23!</math> is a multiple of <math>2^3, 11,</math> and <math>23,</math> we conclude that <math>23!</math> is a multiple of <math>2024.</math> Therefore, we have <math>n=\boxed{\textbf{(D) } 23}.</math> | ||
+ | |||
+ | <u><b>Remark</b></u> | ||
+ | |||
+ | Memorizing the prime factorization of the current year is useful for the AMC 8/10/12 Exams. | ||
+ | |||
+ | ~MRENTHUSIASM | ||
+ | |||
+ | == Video Solution by Pi Academy == | ||
+ | https://youtu.be/GPoTfGAf8bc?si=JYDhLVzfHUbXa3DW | ||
+ | |||
+ | == Video Solution by Daily Dose of Math == | ||
+ | |||
+ | https://youtu.be/DXDJUCVX3yU | ||
+ | |||
+ | ~Thesmartgreekmathdude | ||
+ | |||
+ | == Video Solution 1 by Power Solve == | ||
+ | https://youtu.be/j-37jvqzhrg?si=qwyiAvKLbySyDR7D&t=529 | ||
+ | |||
+ | ==Video Solution by SpreadTheMathLove== | ||
+ | https://www.youtube.com/watch?v=6SQ74nt3ynw | ||
+ | |||
+ | ==See also== | ||
+ | {{AMC10 box|year=2024|ab=A|num-b=4|num-a=6}} | ||
+ | {{AMC12 box|year=2024|ab=A|num-b=3|num-a=5}} | ||
+ | {{MAA Notice}} |
Latest revision as of 08:55, 15 November 2024
- The following problem is from both the 2024 AMC 10A #5 and 2024 AMC 12A #4, so both problems redirect to this page.
Contents
Problem
What is the least value of such that is a multiple of ?
Solution
Note that in the prime factorization. Since is a multiple of and we conclude that is a multiple of Therefore, we have
Remark
Memorizing the prime factorization of the current year is useful for the AMC 8/10/12 Exams.
~MRENTHUSIASM
Video Solution by Pi Academy
https://youtu.be/GPoTfGAf8bc?si=JYDhLVzfHUbXa3DW
Video Solution by Daily Dose of Math
~Thesmartgreekmathdude
Video Solution 1 by Power Solve
https://youtu.be/j-37jvqzhrg?si=qwyiAvKLbySyDR7D&t=529
Video Solution by SpreadTheMathLove
https://www.youtube.com/watch?v=6SQ74nt3ynw
See also
2024 AMC 10A (Problems • Answer Key • Resources) | ||
Preceded by Problem 4 |
Followed by Problem 6 | |
1 • 2 • 3 • 4 • 5 • 6 • 7 • 8 • 9 • 10 • 11 • 12 • 13 • 14 • 15 • 16 • 17 • 18 • 19 • 20 • 21 • 22 • 23 • 24 • 25 | ||
All AMC 10 Problems and Solutions |
2024 AMC 12A (Problems • Answer Key • Resources) | |
Preceded by Problem 3 |
Followed by Problem 5 |
1 • 2 • 3 • 4 • 5 • 6 • 7 • 8 • 9 • 10 • 11 • 12 • 13 • 14 • 15 • 16 • 17 • 18 • 19 • 20 • 21 • 22 • 23 • 24 • 25 | |
All AMC 12 Problems and Solutions |
The problems on this page are copyrighted by the Mathematical Association of America's American Mathematics Competitions.